Averaging measurement with stat +sys errors

Click For Summary
The discussion centers on calculating the average of two measurements with correlated systematic uncertainties. The covariance matrix is derived from both statistical and systematic errors, leading to an inverse matrix that produces unexpected weights for the measurements. Specifically, the weights result in w_1 = 1 and w_2 = 0, indicating that the second measurement contributes nothing to the average, which raises concerns about the validity of the averaging process. The issue is attributed to the determinant being zero, suggesting the use of a pseudoinverse matrix. The conclusion emphasizes that in cases of 100% correlated systematic uncertainties, not averaging may be the best approach.
ChrisVer
Science Advisor
Messages
3,372
Reaction score
465

Homework Statement



You make a measurement of two variables with 100% correlated systematic uncertainty:
x_1 \pm \Delta x_1^{stat} \pm \Delta x_1^{sys} = 1.0 \pm 0.1 \pm 0.1
x_2 \pm \Delta x_2^{stat} \pm \Delta x_2^{sys} = 1.2 \pm 0.1 \pm 0.2

The average is taken by:

\bar{x} = \sum_{i=1}^2 w_i x_i

where w_i = \frac{\sum_j (C^{-1})_{ij}}{ \sum_{kj} (C^{-1})_{kj}} and C=C^{stat}+ C^{sys} the covariance matrix of the measurement.

Homework Equations



All given above

The Attempt at a Solution



I calculate C to get its inverse and find the weights.
For that I deduced that:
C^{stat} = \begin{pmatrix} (\sigma^{stat}_1)^2 & 0 \\ 0 & (\sigma_2^{stat})^2 \end{pmatrix}
and
C^{sys} =\begin{pmatrix} (\sigma^{sys}_1)^2 & \sigma^{sys}_1 \sigma^{sys}_2 \\ \sigma^{sys}_1 \sigma^{sys}_2 & (\sigma_2^{sys})^2 \end{pmatrix}
due to the 100% correlated systematic uncertainties \sigma_{12}^{sys} = \rho \sigma_1^{sys} \sigma_2^{sys}= \sigma_1^{sys} \sigma_2^{sys}.

When I go to get C then:

C=C^{stat} +C^{sys}= \begin{pmatrix} 0.01 & 0 \\ 0 & 0.01 \end{pmatrix} +\begin{pmatrix} 0.01 & 0.02 \\ 0.02 & 0.04 \end{pmatrix} =\frac{1}{100} \begin{pmatrix}2 & 2 \\ 2 & 5 \end{pmatrix}

The inverse of this matrix is C^{-1} = \frac{50}{3} \begin{pmatrix} 5 & -2 \\ -2 & 2 \end{pmatrix}.

My problem is that with such a matrix I am getting for the weights:
w_1 =\frac{\sum_j (C^{-1})_{1j}}{ \sum_{kj} (C^{-1})_{kj}}= \dfrac{\frac{50}{3} (5-2)}{ \frac{50}{3}(5+2-2-2)}= 1

And
w_2 = 0 (since C_{21}^{-1}= - C_{22}^{-1}).

I don't know why this is happening... Any idea?
Obviously this doesn't seem to make sense because in the averaging I won't get any contribution from x_2...
 
Last edited:
Physics news on Phys.org
Including the second measurement blows up the systematic error without reducing the statistical error much. To check this, you can give the second measurement the weight ##w_2 = \epsilon \ll 1## and see what the combined uncertainty is (compared to w2=0).
I can imagine that not averaging at all is the best you can do in this special case where the systematics are weird (100% correlated, but much larger in the second case).
 
The thing is that this makes it a bit more strange... Because I tried before doing the same for x_1= 0.1 \pm 0.0 \pm 0.1 and x_2= 1.0 \pm 0.0 \pm 0.2 (no statistical error). The covariance matrix was:
C= \frac{1}{100} \begin{pmatrix} 1 & 2 \\ 2 & 4 \end{pmatrix} \Rightarrow C^{-1} = \begin{pmatrix} 4 & 8 \\ 8 & 16 \end{pmatrix}
And the weigths were found to be w_1= \frac{1}{3} and w_2= \frac{2}{3} which make sense...

I will try to work out with w_2= \epsilon \ll 1 then... do you think w_1 = 1 - \epsilon as well?
 
Also that's a weird inverse, since [C^{-1} C ]_{11}= \frac{1}{100} (4+16) \ne 1...

*edit and just realized that the determinant is zero and wolfram was giving me a pseudoinverse matrix*
 
ChrisVer said:
Also that's a weird inverse, since [C^{-1} C ]_{11}= \frac{1}{100} (4+16) \ne 1...

*edit and just realized that the determinant is zero and wolfram was giving me a pseudoinverse matrix*
Ah, that could be the problem.

Without statistical errors the weights should certainly be 1 and 0, as using the value with the larger (but 100% correlated) systematics is pointless.

##1-\epsilon## for the other weight, sure.
 
Question: A clock's minute hand has length 4 and its hour hand has length 3. What is the distance between the tips at the moment when it is increasing most rapidly?(Putnam Exam Question) Answer: Making assumption that both the hands moves at constant angular velocities, the answer is ## \sqrt{7} .## But don't you think this assumption is somewhat doubtful and wrong?

Similar threads

  • · Replies 6 ·
Replies
6
Views
1K
  • · Replies 6 ·
Replies
6
Views
2K
  • · Replies 3 ·
Replies
3
Views
2K
Replies
4
Views
2K
  • · Replies 4 ·
Replies
4
Views
2K
  • · Replies 22 ·
Replies
22
Views
2K
  • · Replies 12 ·
Replies
12
Views
2K
  • · Replies 8 ·
Replies
8
Views
1K
  • · Replies 11 ·
Replies
11
Views
2K
Replies
6
Views
2K